[Rozgrzewka OM][MIX][Nierówności] Nierówności

Zadania z kółek matematycznych lub obozów przygotowujących do OM. Problemy z minionych olimpiad i konkursów matematycznych.
Regulamin forum
Wszystkie tematy znajdujące się w tym dziale powinny być tagowane tj. posiadać przedrostek postaci [Nierówności], [Planimetria], itp.. Temat może posiadać wiele różnych tagów. Nazwa tematu nie może składać się z samych tagów.
Awatar użytkownika
timon92
Użytkownik
Użytkownik
Posty: 1654
Rejestracja: 6 paź 2008, o 16:47
Płeć: Mężczyzna
Lokalizacja: Katowice
Podziękował: 7 razy
Pomógł: 472 razy

Re: [Rozgrzewka OM][MIX][Nierówności] Nierówności

Post autor: timon92 »

pozwolę sobie przepisać jedno z oficjalnych rozwiązań

Rozważmy \(n \times n\)-macierz \(A\) o wyrazach \(a_{i,j} = \sqrt{|x_i+x_j|} - \sqrt{|x_i-x_j|}\). Należy wykazać, że \(\vec e^TA\vec e \ge 0\) gdzie \(\vec e\) jest wektorem "jedynkowym". Zamiast tego udowodnimy, że ta nierówność zachodzi dla dowolnego wektora \(\vec e \in \mathbb R^n\), czyli że macierz \(A\) jest dodatnio półokreślona.

W tym celu wystarczy znaleźć prehilbertowską przestrzeń \(V\) i przekształcenie \(f\colon \mathbb R \to V\) takie, że \(a_{i,j}=\langle f(x_i), f(x_j)\rangle\). W rzeczy samej, to jest równoważne dodatniej półokreśloności \(A\). Jednym z możliwych wyborów przestrzeni \(V\) i przekształcenia \(f\) są \(V = L_2(\mathbb R_+)\) i \(f(x)=c \cdot \dfrac{\sin(xt)}{t^{3/4}} \in L_2(\mathbb R_+)\) dla pewnej konkretnej dodatniej stałej \(c\). By udowodnić równość \(a_{i,j} = \langle f(x_i), f(x_j)\rangle\), rozważmy następującą całkę zależną od rzeczywistego parametru \(p\): $$I(p) = \int_0^\infty \frac{1-\cos(px)}{x\sqrt x} \dd x,$$
która oczywiście jest zbieżna do liczby dodatniej. Przez zamianę zmiennej \(y=|p|x\) widzimy, że \(I(p)=\sqrt{|p|}I(1)\). Zatem, korzystając ze wzoru \(\cos(\alpha-\beta)-\cos(\alpha+\beta)=2\sin\alpha\sin\beta\), otrzymujemy $$\sqrt{|a+b|}-\sqrt{|a-b|} = \frac{1}{I(1)}\int_0^\infty \frac{\cos((a-b)x)-\cos((a+b)x)}{x\sqrt x} \dd x = \frac{1}{I(1)} \int_0^\infty \frac{2\sin(ax)\sin(bx)}{x\sqrt x} \dd x,$$
co oczywiście daje \(a_{i,j}=\langle f(x_i),f(x_j)\rangle\) dla \(c=\sqrt{2/I(1)}\)

------------------------

nowe zadanie: dane są liczby \(a\ge b\ge c\ge d\ge 1\) spełniające \(bc<ad\)

rozstrzygnąć, która liczba jest większa: \(a^{2021}+b^{2022}+c^{2022}+d^{2021}\) czy \(a^{2022}+b^{2021}+c^{2021}+d^{2022}\)
Awatar użytkownika
Premislav
Użytkownik
Użytkownik
Posty: 15685
Rejestracja: 17 sie 2012, o 13:12
Płeć: Mężczyzna
Lokalizacja: Warszawa
Podziękował: 195 razy
Pomógł: 5219 razy

Re: [Rozgrzewka OM][MIX][Nierówności] Nierówności

Post autor: Premislav »

Ukryta treść:    
Dodano po 37 minutach 15 sekundach:
No dobra, użycie tutaj Karamaty było przesadą nawet jak na mnie, nic dziwnego, że cokolwiek robię, przegrywam.
wersja elementarna:    
a4karo
Użytkownik
Użytkownik
Posty: 22174
Rejestracja: 15 maja 2011, o 20:55
Płeć: Mężczyzna
Lokalizacja: Bydgoszcz
Podziękował: 38 razy
Pomógł: 3748 razy

Re: [Rozgrzewka OM][MIX][Nierówności] Nierówności

Post autor: a4karo »

Dowód z Karamatą jest ładniejszy (i idzie również w ogólnym przypadku).

Pytanie o Wolfram pozostaje otwarte.
Awatar użytkownika
Premislav
Użytkownik
Użytkownik
Posty: 15685
Rejestracja: 17 sie 2012, o 13:12
Płeć: Mężczyzna
Lokalizacja: Warszawa
Podziękował: 195 razy
Pomógł: 5219 razy

Re: [Rozgrzewka OM][MIX][Nierówności] Nierówności

Post autor: Premislav »

Widocznie wolfram coś źle liczy, bądź, co bardziej prawdopodobne, karmię go złymi syntaktycznie napisami; rzadko używam Karamaty, więc myślałem, że mogłem coś zepsuć (ale teraz nie sądzę), ale w drugiej metodzie niczego nie zepsułem; to jeszcze potrafię ocenić.

Tymczasem podrzucam kolejne zadanko.

Niech \(\displaystyle{ a_1, a_2\ldots a_n>0}\), zaś \(\displaystyle{ n\ge 2}\). Proszę wykazać, że
\(\displaystyle{ \sum_{i=1}^n\max\left\{a_1, a_{2}\ldots a_i\right\}\cdot \min \left\{a_i, a_{i+1}, \ldots a_n\right\}\le \frac{n}{2\sqrt{n-1}}\sum_{i=1}^na_i^2}\).
Awatar użytkownika
timon92
Użytkownik
Użytkownik
Posty: 1654
Rejestracja: 6 paź 2008, o 16:47
Płeć: Mężczyzna
Lokalizacja: Katowice
Podziękował: 7 razy
Pomógł: 472 razy

Re: [Rozgrzewka OM][MIX][Nierówności] Nierówności

Post autor: timon92 »

masz jakieś zgrabniejsze rozwiązanie od poniższego?
Ukryta treść:    
nowe :arrow: \(a,b,c>0, a+b+c=3 \implies (a^2-ab+b^2)(b^2-bc+c^2)(c^2-ca+a^2)\le 12\)
Awatar użytkownika
Premislav
Użytkownik
Użytkownik
Posty: 15685
Rejestracja: 17 sie 2012, o 13:12
Płeć: Mężczyzna
Lokalizacja: Warszawa
Podziękował: 195 razy
Pomógł: 5219 razy

Re: [Rozgrzewka OM][MIX][Nierówności] Nierówności

Post autor: Premislav »

Ja nie umiałem w ogóle udowodnić poprzedniej nierówności, ale uznałem, że jest dość nietypowa i chciałbym poznać dowód. To są Chiny Zachodnie 2017, zadanie 8 (wygooglować ładniejszego rozwiązania też mi się nie udało, ale to pewnie moja nieudolność, choć nie uważam, żeby Twoje było jakkolwiek niezgrabne).
bieżące:    
Coś za łatwo poszło, więc poczekam z wrzuceniem następnego zadania.
Awatar użytkownika
timon92
Użytkownik
Użytkownik
Posty: 1654
Rejestracja: 6 paź 2008, o 16:47
Płeć: Mężczyzna
Lokalizacja: Katowice
Podziękował: 7 razy
Pomógł: 472 razy

Re: [Rozgrzewka OM][MIX][Nierówności] Nierówności

Post autor: timon92 »

solw jest wporzo, czekamy na nową nierówność :!:
Awatar użytkownika
Premislav
Użytkownik
Użytkownik
Posty: 15685
Rejestracja: 17 sie 2012, o 13:12
Płeć: Mężczyzna
Lokalizacja: Warszawa
Podziękował: 195 razy
Pomógł: 5219 razy

Re: [Rozgrzewka OM][MIX][Nierówności] Nierówności

Post autor: Premislav »

Liczby rzeczywiste dodatnie \(\displaystyle{ a_1, a_2\ldots a_n}\) spełniają zależność
\(\displaystyle{ \sum_{i=1}^n\frac{1}{1+a_i}\le 1}\). Proszę wykazać, że
\(\displaystyle{ \sum_{i=1}^n\frac{1}{2^{a_i}}\le 1}\).
Dragomier
Użytkownik
Użytkownik
Posty: 4
Rejestracja: 5 paź 2020, o 19:48
Płeć: Mężczyzna
wiek: 17

Re: [Rozgrzewka OM][MIX][Nierówności] Nierówności

Post autor: Dragomier »

Teza nierówności wynika z nierówności Bernoulliego dla \(\displaystyle{ x=1}\) i \(\displaystyle{ n= a_n.}\)
Ostatnio zmieniony 5 lip 2022, o 02:04 przez Jan Kraszewski, łącznie zmieniany 1 raz.
Powód: Używaj LaTeXa do wszystkich wyrażeń matematycznych.
Awatar użytkownika
Premislav
Użytkownik
Użytkownik
Posty: 15685
Rejestracja: 17 sie 2012, o 13:12
Płeć: Mężczyzna
Lokalizacja: Warszawa
Podziękował: 195 razy
Pomógł: 5219 razy

Re: [Rozgrzewka OM][MIX][Nierówności] Nierówności

Post autor: Premislav »

Dragomier pisze: 5 lip 2022, o 00:21 Teza nierówności wynika z nierówności Bernoulliego dla \(\displaystyle{ x=1}\) i \(\displaystyle{ n= a_n.}\)
Niestety nie. :) Taki był też mój pierwszy odruch, rzecz w tym, że w nierówności Bernoulliego mamy przeciwny zwrot, gdy zdarzy się, że któreś z \(\displaystyle{ a_i}\) jest nie większe niż \(\displaystyle{ 1}\).
Tzn. nierówność Bernoulliego w klasycznej formie wygląda tak:
dla dowolnego \(\displaystyle{ x>-1}\) i dowolnego \(\displaystyle{ a\ge 1}\) zachodzi \(\displaystyle{ (1+x)^a\ge 1+ax}\).
Jeśli natomiast \(\displaystyle{ a\in(0,1)}\), to zachodzi nierówność z przeciwnym zwrotem.

Ale skoro zadanie tak długo wisi, to wrzucę wzorcówkę z Kurlandczyka (Powrót do krainy nierówności, zadanie 6.1.16*), bo moje rozwiązanie było dużo brzydsze i używało niemałej dawki analizy matematycznej (a że ja i studia to zbiory rozłączne od kilku lat, to nie dam głowy, czy poprawne).
Ukryta treść:    
Chyba nie mam wyczucia do zadań, więc następny problemat może zaproponować dowolna osoba.
Awatar użytkownika
mol_ksiazkowy
Użytkownik
Użytkownik
Posty: 11266
Rejestracja: 9 maja 2006, o 12:35
Płeć: Mężczyzna
Lokalizacja: Kraków
Podziękował: 3143 razy
Pomógł: 747 razy

Re: [Rozgrzewka OM][MIX][Nierówności] Nierówności

Post autor: mol_ksiazkowy »

:arrow: Czy jeśli ciąg \(\displaystyle{ x_n}\) jest taki, że \(\displaystyle{ |x_{m+n} - x_m- x_n | < \frac{1}{m+n}}\) dla dowolnych \(\displaystyle{ m, n }\) (liczby naturalne), to znaczy, że jest to ciąg arytmetyczny ?

Źródło: Kwant
Awatar użytkownika
Premislav
Użytkownik
Użytkownik
Posty: 15685
Rejestracja: 17 sie 2012, o 13:12
Płeć: Mężczyzna
Lokalizacja: Warszawa
Podziękował: 195 razy
Pomógł: 5219 razy

Re: [Rozgrzewka OM][MIX][Nierówności] Nierówności

Post autor: Premislav »

Ukryta treść:    
a4karo
Użytkownik
Użytkownik
Posty: 22174
Rejestracja: 15 maja 2011, o 20:55
Płeć: Mężczyzna
Lokalizacja: Bydgoszcz
Podziękował: 38 razy
Pomógł: 3748 razy

Re: [Rozgrzewka OM][MIX][Nierówności] Nierówności

Post autor: a4karo »

Premislav pisze: 12 lip 2022, o 14:28
Ukryta treść:    
Ciąg `y_n` jest zdeterminowany ciągiem `x_n` więc nie za bardzo możemy nim manipulować
Awatar użytkownika
Premislav
Użytkownik
Użytkownik
Posty: 15685
Rejestracja: 17 sie 2012, o 13:12
Płeć: Mężczyzna
Lokalizacja: Warszawa
Podziękował: 195 razy
Pomógł: 5219 razy

Re: [Rozgrzewka OM][MIX][Nierówności] Nierówności

Post autor: Premislav »

a4karo pisze: 12 lip 2022, o 23:27

Ciąg `y_n` jest zdeterminowany ciągiem `x_n` więc nie za bardzo możemy nim manipulować
Formalnie racja, ale uważam to już za lekkie czepialstwo. Ciąg \(\displaystyle{ x_n}\) jest arytmetyczny wtedy i tylko wtedy, gdy \(\displaystyle{ -x_n}\) jest arytmetyczny, a skoro określimy \(\displaystyle{ x_n=nx_1+y_n}\), to \(\displaystyle{ -x_n=n\cdot (-x_1)-y_n}\).
Zresztą, \(\displaystyle{ y_k>0}\) nie jest takie istotne, wystarczy w założeniu nie wprost \(\displaystyle{ |y_k|>0}\), wtenczas - skoro \(\displaystyle{ \lim_{n\to\infty}(y_{n+k}-y_n)=y_k}\), to \(\displaystyle{ \lim_{n\to \infty}|y_{n+k}-y_n|=|y_k|}\) i reszta idzie analogicznie, z \(\displaystyle{ \varepsilon=\frac{|y_k|}{2}}\).

Dodano po 13 minutach 13 sekundach:
Uważam, że rozwiązałem zadanie (w razie wątpliwości zapraszam do ich zgłaszania), więc macie nowe:
niech \(\displaystyle{ a,b,c\in [0,1]}\). Proszę udowodnić, że
\(\displaystyle{ \frac{a}{b+c+1}+\frac{b}{c+a+1}+\frac c{a+b+1}+(1-a)(1-b)(1-c)\le 1}\).
a4karo
Użytkownik
Użytkownik
Posty: 22174
Rejestracja: 15 maja 2011, o 20:55
Płeć: Mężczyzna
Lokalizacja: Bydgoszcz
Podziękował: 38 razy
Pomógł: 3748 razy

Re: [Rozgrzewka OM][MIX][Nierówności] Nierówności

Post autor: a4karo »

To ja jeszcze wrócę do poprzedniego:
Ukryta treść:    
ODPOWIEDZ